Skip to main content
Log in

A restricted nonlocal operator bridging together the Laplacian and the fractional Laplacian

  • Published:
Calculus of Variations and Partial Differential Equations Aims and scope Submit manuscript

Abstract

In this work we introduce volume constraint problems involving the nonlocal operator \((-\Delta )_{\delta }^{s}\), closely related to the fractional Laplacian \((-\Delta )^{s}\), and depending upon a parameter \(\delta >0\) called horizon. We study the associated linear and spectral problems and the behavior of these volume constraint problems when \(\delta \rightarrow 0^+\) and \(\delta \rightarrow +\infty \). Through these limit processes on \((-\Delta )_{\delta }^{s}\) we derive spectral convergence to the local Laplacian and to the fractional Laplacian as \(\delta \rightarrow 0^+\) and \(\delta \rightarrow +\infty \) respectively, as well as we prove the convergence of solutions of these problems to solutions of a local Dirichlet problem involving \((-\Delta )\) as \(\delta \rightarrow 0^+\) or to solutions of a nonlocal fractional Dirichlet problem involving \((-\Delta )^s\) as \(\delta \rightarrow +\infty \).

This is a preview of subscription content, log in via an institution to check access.

Access this article

Price excludes VAT (USA)
Tax calculation will be finalised during checkout.

Instant access to the full article PDF.

Similar content being viewed by others

Notes

  1. Since \(\{\mathcal {D}_{\delta }\}_{\delta \ge 0}\) is monotone increasing we have \(\liminf \nolimits _{\delta \rightarrow +\infty } \mathcal {D}_{\delta }=\bigcup \nolimits _{\delta \ge 0}\bigcap \nolimits _{\mu \ge \delta } \mathcal {D}_{\mu }=\bigcup \nolimits _{\delta \ge 0}\mathcal {D}_{\delta }\) as well as \(\limsup \nolimits _{\delta \rightarrow +\infty } \mathcal {D}_{\delta }=\bigcap \nolimits _{\delta \ge 0}\bigcup \nolimits _{\mu \ge \delta } \mathcal {D}_{\mu }=\bigcap \nolimits _{\delta \ge 0}\bigcup \nolimits _{\mu \ge 0} \mathcal {D}_{\mu }=\bigcup \nolimits _{\delta \ge 0}\mathcal {D}_{\delta }\). Then, \(\lim \nolimits _{\delta \rightarrow +\infty }\mathcal {D}_{\delta }= \bigcup \nolimits _{\delta \ge 0}\mathcal {D}_{\delta }=\mathcal {D}\).

References

  1. Alali, B., Albin, N.: Fourier multipliers for nonlocal Laplace operators. Appl. Anal. 1, 1–21 (2019)

    Article  Google Scholar 

  2. Alberti, G., Bellettini, G.: A non-local anisotropic model for phase transitions: asymptotic behaviour of rescaled energies. Eur. J. Appl. Math. 9, 261–284 (1998)

    Article  MathSciNet  Google Scholar 

  3. Andrés, F., Muñoz, J.: A type of nonlocal elliptic problem: existence and approximation through a Galerkin–Fourier method. SIAM J. Math. Anal. 47, 498–525 (2015)

    Article  MathSciNet  Google Scholar 

  4. Andreu-Vaillo, F., Mazón, J.M., Rossi, J.D., Toledo-Melero, J.J.: Nonlocal diffusion problems. Mathematical Surveys and Monographs, American Mathematical Society, Providence, vol. 165. RI; Real Sociedad Matemática Española, Madrid (2010)

  5. Bellido, J.C., Mora-Corral, C.: Existence for nonlocal variational problems in peridynamics. SIAM J. Math. Anal. 46, 890–916 (2014)

    Article  MathSciNet  Google Scholar 

  6. Bellido, J.C., Mora-Corral, C., Pedregal, P.: Hyperelasticity as a \(\Gamma \)-limit of peridynamics when the horizon goes to zero. Calc. Var. Part. Differ. Equ. 54, 1643–1670 (2015)

    Article  MathSciNet  Google Scholar 

  7. Boulanger, J., Elbau, P., Pontow, C., Scherzer, O.: Non-local functionals for imaging. In: Fixed-point Algorithms for Inverse Problems in Science and Engineering, vol. 49 of Springer Optim. Appl. Springer, New York, pp. 131–154 (2011)

  8. Bourgain, J., Brezis, H., Mironescu, P.: Another look at Sobolev spaces, in Optimal control and partial differential equations, pp. 439–455. IOS, Amsterdam (2001)

    MATH  Google Scholar 

  9. Braides, A.: \(\Gamma \)-convergence for beginners. Oxford Lecture Series in Mathematics and its Applications, vol. 22. Oxford University Press, Oxford (2002)

  10. Brasco, L., Parini, E., Squassina, M.: Stability of variational eigenvalues for the fractional \(p\)-Laplacian. Discret. Contin. Dyn. Syst. 36, 1813–1845 (2016)

    Article  MathSciNet  Google Scholar 

  11. Brezis, H.: Functional analysis, Sobolev spaces and partial differential equations. Universitext, Springer, New York (2011)

    MATH  Google Scholar 

  12. Bucur, C.: Some observations on the Green function for the ball in the fractional Laplace framework. Commun. Pure Appl. Anal. 15, 657–699 (2016)

    Article  MathSciNet  Google Scholar 

  13. Bucur, C., Valdinoci, E.: Nonlocal diffusion and applications. Lecture Notes of the Unione Matematica Italiana, Springer, [Cham], vol. 20. Unione Matematica Italiana, Bologna (2016)

  14. D’Elia, M., Gunzburger, M.: The fractional Laplacian operator on bounded domains as a special case of the nonlocal diffusion operator. Comput. Math. Appl. 66, 1245–1260 (2013)

    Article  MathSciNet  Google Scholar 

  15. Di Nezza, E., Palatucci, G., Valdinoci, E.: Hitchhiker’s guide to the fractional Sobolev spaces. Bull. Sci. Math. 136, 521–573 (2012)

    Article  MathSciNet  Google Scholar 

  16. Du, Q.: Nonlocal modeling, analysis, and computation. CBMS-NSF Regional Conference Series in Applied Mathematics. Society for Industrial and Applied Mathematics (SIAM), vol. 94., Philadelphia PA (2019)

  17. Duo, S., Wang, H., Zhang, Y.: A comparative study on nonlocal diffusion operators related to the fractional Laplacian. Discret. Contin. Dyn. Syst. Ser. B 24, 231–256 (2019)

    MathSciNet  MATH  Google Scholar 

  18. Evgrafov, A., Bellido, J.C.: From non-local Eringen’s model to fractional elasticity. Math. Mech. Solids 24, 1935–1953 (2019)

    Article  MathSciNet  Google Scholar 

  19. Gilboa, G., Osher, S.: Nonlocal operators with applications to image processing. Multisc. Model. Simul. 7, 1005–1028 (2008)

    Article  MathSciNet  Google Scholar 

  20. Kassmann, M., Mengesha, T., Scott, J.: Solvability of nonlocal systems related to peridynamics. Commun. Pure Appl. Anal. 18, 1303–1332 (2019)

    Article  MathSciNet  Google Scholar 

  21. Kindermann, S., Osher, S., Jones, P.W.: Deblurring and denoising of images by nonlocal functionals. Multiscale Model. Simul. 4, 1091–1115 (2005)

    Article  MathSciNet  Google Scholar 

  22. Leoni, G., Spector, D.: Characterization of Sobolev and \(BV\) spaces. J. Funct. Anal. 261, 2926–2958 (2011)

    Article  MathSciNet  Google Scholar 

  23. Lions, J.-L., Magenes, E.: Non-homogeneous boundary value problems and applications. Vol. I, Springer-Verlag, New York-Heidelberg, 1972. Translated from the French by P. Kenneth, Die Grundlehren der mathematischen Wissenschaften, Band 181 (1972)

  24. Lischke, A., Pang, G., Gulian, M., et al.: What is the fractional Laplacian? A comparative review with new results. J. Comput. Phys. 404, 109009 (2020)

    Article  MathSciNet  Google Scholar 

  25. Mengesha, T.: Nonlocal Korn-type characterization of Sobolev vector fields. Commun. Contemp. Math. 14, 1250028 (2012)

    Article  MathSciNet  Google Scholar 

  26. Mengesha, T., Du, Q.: Characterization of function spaces of vector fields and an application in nonlinear peridynamics. Nonlinear Anal. 140, 82–111 (2016)

    Article  MathSciNet  Google Scholar 

  27. Mengesha, T., Spector, D.: Localization of nonlocal gradients in various topologies. Calc. Var. Part. Differ. Equ. 52, 253–279 (2015)

    Article  MathSciNet  Google Scholar 

  28. Ponce, A.C.: A new approach to Sobolev spaces and connections to \(\Gamma \)-convergence. Calc. Var. Part. Differ. Equ. 19, 229–255 (2004)

    Article  MathSciNet  Google Scholar 

  29. Ros-Oton, X.: Nonlocal elliptic equations in bounded domains: a survey. Publ. Mat. 60, 3–26 (2016)

    Article  MathSciNet  Google Scholar 

  30. Ros-Oton, X., Serra, J.: Fractional Laplacian: Pohozaev identity and nonexistence results. Compte. R. Mathematique 350, 505–508 (2012)

    Article  MathSciNet  Google Scholar 

  31. Servadei, R., Valdinoci, E.: A Brezis-Nirenberg result for non-local critical equations in low dimension. Commun. Pure Appl. Anal. 12, 2445–2464 (2013)

    Article  MathSciNet  Google Scholar 

  32. Servadei, R., Valdinoci, E.: Mountain pass solutions for non-local elliptic operators. J. Math. Anal. Appl. 389, 887–898 (2012)

    Article  MathSciNet  Google Scholar 

  33. Servadei, R., Valdinoci, E.: Variational methods for non-local operators of elliptic type. Discret. Contin. Dyn. Syst. 33, 2105–2137 (2013)

    Article  MathSciNet  Google Scholar 

  34. Servadei, R., Valdinoci, E.: On the spectrum of two different fractional operators. Proc. R. Soc. Edinb.: Sect. Math. 144, 831–855 (2014)

    Article  MathSciNet  Google Scholar 

  35. Silling, S.A.: Reformulation of elasticity theory for discontinuities and long-range forces. J. Mech. Phys. Solids 48, 175–209 (2000)

    Article  MathSciNet  Google Scholar 

  36. Spector, D.: On a generalization of \(L^p\)-differentiability. Calc. Var. Part. Differ. Equ. 55, 21 (2016)

    Article  Google Scholar 

  37. Stinga, P.R., Torrea, J.L.: Extension problem and Harnack’s inequality for some fractional operators. Comm. Part. Differ. Equ. 35, 2092–2122 (2010)

    Article  MathSciNet  Google Scholar 

Download references

Author information

Authors and Affiliations

Authors

Corresponding author

Correspondence to Alejandro Ortega.

Additional information

Communicated by A. Malchiodi.

Publisher's Note

Springer Nature remains neutral with regard to jurisdictional claims in published maps and institutional affiliations.

This work has been supported by the Agencia Estatal de Investigación, Ministerio de Ciencia e Innovación (Spain) through project MTM2017-83740-P. This paper is in final form and no version of it will be submitted for publication elsewhere.

Appendix A: Proof of secondary results

Appendix A: Proof of secondary results

Proof of Lemma 2

It is immediate to see that \(\langle \cdot ,\cdot \rangle _{\mathbb {H}_0^{\delta ,s}}\) is a scalar product on \(\mathbb {H}_0^{\delta ,s}(\Omega )\times \mathbb {H}_0^{\delta ,s}(\Omega )\). Let us see that \(\mathbb {H}_0^{\delta ,s}(\Omega )\) is complete with respect to the norm . Let \(\{v_n\}_{n\in \mathbb {N}}\) be a Cauchy sequence in \(\mathbb {H}_0^{\delta ,s}(\Omega )\). Hence, given \(varepsilon>0\), and keeping in mind Proposition 1 and Lemma 1, there exist \(c>0\) and \(n_0=n_0(\varepsilon )\in \mathbb {N}\) such that, for \(m,n\ge n_0\), we have

(A.1)

and consequently \(\{v_n\}_{n\in \mathbb {N}}\) is also a Cauchy sequence in \(L^2(\Omega )\), and since this is a complete space, there exists \(v\in L^2(\Omega )\) such that \(v_n\rightarrow v\) in \(L^2(\Omega )\) as \(n\rightarrow +\infty \). Arguing as in [32, Lemma 7], it is easily proved that \(v\in \mathbb {H}_0^{\delta ,s}(\Omega )\) and that \(\{v_n\}_{n\in \mathbb {N}}\) converges in \(\mathbb {H}_0^{\delta ,s}(\Omega )\). Hence, the space \(\mathbb {H}_0^{\delta ,s}(\Omega )\) is a complete space. \(\square \)

Proof of Lemma 3

Since the sequence \(\{v_j\}_{j\in \mathbb {N}}\) is bounded in \(\mathbb {H}_0^{\delta ,s}(\Omega )\), because of Proposition 1, it is bounded in \(H^s(\Omega _{\delta })\) so that it is also bounded in \(H^s(\Omega )\) and, hence, in \(L^2(\Omega )\). Observe that, although \(v=0\) on \(\partial _{\delta }\Omega \), we have \(\Vert \cdot \Vert _{H^s(\Omega )}\ne \Vert \cdot \Vert _{H^s(\Omega _{\delta })}\) since the latter norm takes into account the interaction between \(\Omega \) and \(\partial _{\delta }\Omega \). Then, by [15, Corollary 7.2], there exists \(v\in L^p(\Omega )\) such that, up to a subsequence,

$$\begin{aligned} v_j\rightarrow v\quad \text {in}\ L^p(\Omega ),\ \text {as}\ j\rightarrow +\infty ,\quad \text {for any }p\in [2,2_s^*). \end{aligned}$$

As mentioned before, the proof of Proposition 2 can be done similarly as to the proof [33, Proposition 9] so that we sketch the main steps.

Let us consider the energy functional

and observe that \(\big \langle \mathcal {J}'(u)\big |v\big \rangle =\frac{c_{N,s}}{2}\langle u,v\rangle _{\mathbb {H}_0^{\delta ,s}}=\big \langle \mathcal {J}'(v)\big | u\big \rangle \) for all \(u,v\in \mathbb {H}_0^{\delta ,s}(\Omega )\).

Lemma 8

Let \(\mathcal {E}\subseteq \mathbb {H}_0^{\delta ,s}(\Omega )\) be a weakly closed subspace and \(\mathcal {E}_1=\{u\in \mathcal {E}:\Vert u\Vert _{L^2(\Omega )}=1\}\). Then, there exists \(u^{\star }\in \mathcal {E}_1\) such that

$$\begin{aligned} \min \limits _{u\in \mathcal {E}_1}\mathcal {J}(u)=\mathcal {J}(u^{\star }). \end{aligned}$$

Moreover,

$$\begin{aligned} \begin{aligned} \langle \mathcal {J}'(u^{\star })|\varphi \rangle&=\frac{c_{N,s}}{2}\int _{\Omega _{\delta }}\int _{\Omega _{\delta }\cap B(x,\delta )}\frac{(u^{\star }(x)-u^{\star }(y))(\varphi (x)-\varphi (y)}{|x-y|^{N+2s}}dydx\\&=\lambda ^{\star }\int _{\Omega }u^{\star }(x)\varphi (x)dx, \end{aligned} \end{aligned}$$

for all \(\varphi \in \mathcal {E}\) and

(A.2)

Proof

Let \(\{u_j\}\subset \mathcal {E}_1\) be a minimizing sequence for \(\mathcal {J}\), i.e.,

$$\begin{aligned} \mathcal {J}(u_j)\rightarrow \inf \limits _{u\in \mathcal {E}_1}\mathcal {J}(u)\ge 0. \end{aligned}$$
(A.3)

Hence, the sequence \(\mathcal {J}(u_j)\) is bounded in \(\mathbb {R}\) and by definition of \(\mathcal {J}\), the sequence is also bounded. Moreover, because of Lemma 2 the space \(\mathbb {H}_0^{\delta ,s}(\Omega )\) is a Hilbert space and, hence, it is also a reflexive space. Then, up to a subsequence (that we do nor relabel), \(u_j\) converges weakly in \(\mathbb {H}_0^{\delta ,s}(\Omega )\) to some \(u^{\star }\in \mathcal {E}\) since \(\mathcal {E}\) is weakly closed. Therefore, we conclude

$$\begin{aligned} \big \langle \mathcal {J}'(u_j)\big |\varphi \big \rangle \rightarrow \big \langle \mathcal {J}'(u^{\star })\big |\varphi \big \rangle \quad \text {for all}\ \varphi \in \mathbb {H}_0^{\delta ,s}(\Omega ). \end{aligned}$$

On the other hand, since is bounded, because of Lemma 3, we have

$$\begin{aligned} u_j\rightarrow u^{\star }\ \text {in}\ L^2(\Omega ), \end{aligned}$$

so that \(\Vert u^{\star }\Vert _{L^2(\Omega )}=1\) and then \(u^{\star }\in \mathcal {E}_1\). Observe that the functional \(\mathcal {J}\) is continuous and convex in \(\mathbb {H}_0^{\delta ,s}(\Omega )\), so that \(\mathcal {J}\) is weakly lower semicontinuous in \(\mathbb {H}_0^{\delta ,s}(\Omega )\). Then,

$$\begin{aligned} \liminf _{j\rightarrow +\infty }\mathcal {J}(u_j)\ge \mathcal {J}(u^{\star })\ge \inf \limits _{u\in \mathcal {E}_1} \mathcal {J}(u), \end{aligned}$$

and, by (A.3), we conclude \(\mathcal {J}(u^{\star })=\min \limits _{u\in \mathcal {E}_1}\mathcal {J}(u)\). Next, we prove that \(\lambda ^{\star }=\mathcal {J}(u^{\star })\). Let us set \(\varepsilon \in (-1,1)\), \(\varphi \in \mathcal {E}\) and \(u_{\varepsilon }=\frac{u^{\star }+\varepsilon \varphi }{\Vert u^{\star }+\varepsilon \varphi \Vert _{L^2(\Omega )}}\in \mathcal {E}_1\). Then,

Because of the minimality of \(u^{\star }\) we have \(\left. \frac{d}{d\varepsilon }\mathcal {J}(u_{\varepsilon })\right| _{\varepsilon =0}=0\), from where we conclude . \(\square \)

Proof of Proposition 2-(1)

Taking \(\mathcal {E}=\mathbb {H}_0^{\delta ,s}(\Omega )\) in Lemma 8 we find that minimum defining \(\lambda _1^{\delta ,s}\) exists and \(\lambda _1^{\delta ,s}\) is an eigenvalue of problem (\(EP_{\delta }^{s}\)), proving (2.7). Moreover, because of Lemma 8, the minimum defining \(\lambda _1^{\delta ,s}\) is attained at some function \(\varphi _1^{\delta ,s}\in \mathbb {H}_0^{\delta ,s}(\Omega )\) with \(\Vert \varphi _1^{\delta ,s}\Vert _{L^2(\Omega )}=1\). Then, by (A.2), we conclude

proving (2.8). The proof of the non-negativity of the first eigenfunction follows using the triangle inequality and the minimality of \(\varphi _1^{\delta ,s}\) as in [33, Proposition 9-b)]. To prove that \(\lambda _1^{\delta ,s}\) is simple we can argue exactly as in [33, Proposition 9-c)] so we omit the details. \(\square \)

Proof of Proposition 2-(2)

Let us define \(\lambda _k^{\delta ,s}\) by (2.12) and observe that, by Lemma 8 with

$$\begin{aligned} \mathcal {E}=\mathbb {P}_{k}^{\delta }=\{u\in \mathbb {H}_0^{\delta ,s}(\Omega ):\langle u,\varphi _j^{\delta ,s}\rangle _{\mathbb {H}_0^{\delta ,s}}=0,\ \forall j=1,\ldots ,k-1\}, \end{aligned}$$

which is weakly closed by Lemma 2; the minimium in (2.12) exists, it is attained at some function \(\varphi _k^{\delta ,s}\in \mathbb {P}_{k}^{\delta }\) and

$$\begin{aligned} \frac{c_{N,s}}{2}\langle \varphi _{k}^{\delta ,s},\phi \rangle _{\mathbb {H}_0^{\delta ,s}}=\lambda _{k}^{\delta ,s}\int _{\Omega }\varphi _{k}^{\delta ,s}\phi \, dx,\qquad \text {for all }\phi \in \mathbb {P}_{k}^{\delta }. \end{aligned}$$
(A.4)

In order to show that \(\lambda _{k}^{\delta ,s}\) is an eigenvalue associated to the eigenfunction \(\varphi _{k}^{\delta ,s}\), it remains to prove that (A.4) holds for any \(\phi \in \mathbb {H}_0^{\delta ,s}(\Omega )\). This can be done inductively as in [33, Proposition 9-d)]. Note that the initial induction step has been proved in Proposition 2-(1). Let us write

$$\begin{aligned} \mathbb {H}_0^{\delta ,s}(\Omega )=span\{\varphi _1^{\delta ,s},\ldots ,\varphi _k^{\delta ,s}\}\oplus \mathbb {P}_{k+1}^{\delta } \end{aligned}$$

and, given \(\phi \in \mathbb {H}_0^{\delta ,s}(\Omega )\), write \(\phi =\phi _1+\phi _2\) with \(\phi _1=\sum _{i=1}^{k}c_i\varphi _i^{\delta ,s}\) and \(\phi _2\in \mathbb {P}_{k+1}^{\delta }\).

Testing the equation for \(\varphi _{k+1}^{\delta ,s}\) against \(\phi _2=\phi -\phi _1\) and \(\varphi _i^{\delta ,s}\) for \(i=1,\ldots ,k\) it is easily proved that

$$\begin{aligned} \frac{c_{N,s}}{2}\langle \varphi _{k+1}^{\delta ,s},\phi \rangle _{\mathbb {H}_0^{\delta ,s}}-\lambda _{k+1}^{\delta ,s}\int _{\Omega }\varphi _{k+1}^{\delta ,s}\phi \, dx=0, \end{aligned}$$

so that (A.4) holds for any \(\phi \in \mathbb {H}_0^{\delta ,s}(\Omega )\).

We continue by proving (2.10). Since \(\displaystyle \mathbb {P}_{k+1}^{\delta }\subseteq \mathbb {P}_{k}^{\delta }\subseteq \mathbb {H}_0^{\delta ,s}(\Omega )\) we have \(0<\lambda _1^{\delta ,s}\le \lambda _2^{\delta ,s}\le \ldots \le \lambda _{k}^{\delta ,s}\le \ldots \). Moreover, since \(\varphi _2^{\delta ,s}\in \mathbb {P}_{2}^{\delta }\), by Proposition 2-(1) we get \(\lambda _1\ne \lambda _2\) and (2.10) follows. To prove (2.11) let us assume that \(\lambda _{k}^{\delta ,s}\rightarrow c<+\infty \) so that the sequence is bounded in \(\mathbb {H}_0^{\delta ,s}(\Omega )\). Hence, because of Lemma 3 we conclude that, up to a subsequence, \(\varphi _{k}^{\delta ,s}\rightarrow \overline{\varphi }\in \ L^2(\Omega )\) as \(k\rightarrow +\infty \) and \(\{\varphi _{k}^{\delta ,s}\}_{k\in \mathbb {N}}\) is a Cauchy sequence. On the other hand, letting \(k,h\in \mathbb {N}\), \(k>h\), since \(\varphi _{k}^{\delta ,s}\in \mathbb {P}_{k}^{\delta }\) we have that \(\langle \varphi _{k}^{\delta ,s},\varphi _{h}^{\delta ,s}\rangle _{\mathbb {H}_0^{\delta ,s}}=0\). Therefore \(\{\varphi _{k}^{\delta ,s}\}_{k\in \mathbb {N}}\) is an orthogonal sequence in \(\mathbb {H}_0^{\delta ,s}(\Omega )\). Moreover, taking in mind that \(\varphi _{k}^{\delta ,s}\) are eigenfunctions, it follows that

$$\begin{aligned} 0=\frac{c_{N,s}}{2}\langle \varphi _{k}^{\delta ,s},\varphi _{h}^{\delta ,s}\rangle _{\mathbb {H}_0^{\delta ,s}}=\lambda _k^{\delta ,s}\int _{\Omega }\varphi _{k}^{\delta ,s}\varphi _{h}^{\delta ,s}dx, \end{aligned}$$
(A.5)

and we conclude that \(\{\varphi _{k}^{\delta ,s}\}_{k\in \mathbb {N}}\) is also an orthogonal sequence in both \(\mathbb {H}_0^{\delta ,s}(\Omega )\) and \(L^2(\Omega )\). Then, \(\Vert \varphi _k^{\delta ,s}-\varphi _j^{\delta ,s}\Vert _{L^2(\Omega )}^2=\Vert \varphi _k^{\delta ,s}\Vert _{L^2(\Omega )}^2+\Vert \varphi _j^{\delta ,s}\Vert _{L^2(\Omega )}^2=2\) and we get a contradiction with the fact that \(\{\varphi _{k}^{\delta ,s}\}_{k\in \mathbb {N}}\) is a Cauchy sequence. Therefore, \(\lambda _k\rightarrow +\infty \) as \(k\rightarrow +\infty \).

To complete the proof of Proposition 2-(2), it remains to prove that the sequence of eigenvalues given by (2.12) exhust all the eigenvalues of problem (\(EP_{\delta }^{s}\)). This can be done exactly as in [33, Proposition 9-d)] so we omit the details. \(\square \)

Proof of Proposition 2-(3)

The orthogonality of the set \(\{\varphi _k^{\delta ,s}\}_{k\in \mathbb {N}}\) in \(\mathbb {H}_0^{\delta ,s}(\Omega )\) and \(L^2(\Omega )\) follows by (A.5), so it remains to prove that it is a basis for both \(\mathbb {H}_0^{\delta ,s}(\Omega )\) and \(L^2(\Omega )\). This follows using a standard Fourier analysis technique exactly as in [33, Proposition 9-f)] so we omit the details for the sake of brevity. \(\square \)

Proof of Proposition 2-(4)

Let \(n\in \mathbb {N},\ n\ge 1\) and \(\lambda _k^{\delta ,s}>0\) be an eigenvalue satisfying (2.15), i.e.,

$$\begin{aligned} \lambda _{k-1}^{\delta ,s}<\lambda _{k}^{\delta ,s}=\ldots =\lambda _{k+n}^{\delta ,s}<\lambda _{k+n+1}^{\delta ,s}. \end{aligned}$$

Because of Proposition 2-(2), equation (2.12), every function belonging to\(span\{\varphi _k^{\delta ,s},\ldots ,\varphi _{k+n}^{\delta ,s}\}\) is an eigenfunction corresponding to \(\lambda _k^{\delta ,s}=\ldots =\lambda _{k+n}^{\delta ,s}\). It remains to prove that each eigenfunction \(\psi \in \mathbb {H}_0^{\delta ,s}(\Omega )\), \(\psi \ne 0\), associated to \(\lambda _{k}^{\delta ,s}\) belongs to \(span\{\varphi _k^{\delta ,s},\ldots ,\varphi _{k+n}^{\delta ,s}\}\). Let us write

$$\begin{aligned} \mathbb {H}_0^{\delta ,s}(\Omega )=span\{\varphi _k^{\delta ,s},\ldots ,\varphi _{k+n}^{\delta ,s}\}\oplus \left( span\{\varphi _k^{\delta ,s},\ldots ,\varphi _{k+n}^{\delta ,s}\}\right) ^{\bot } \end{aligned}$$

and, consequently, \(\psi =\psi _1+\psi _2\) with the functions \(\psi _1\in span\{\varphi _k^{\delta ,s},\ldots ,\varphi _{k+n}^{\delta ,s}\}\) and \(\psi _2\in \left( span\{\varphi _k^{\delta ,s},\ldots ,\varphi _{k+n}^{\delta ,s}\}\right) ^{\bot }\). Using the orthogonality of the eigenfunctions and arguing as in [33, Proposition 9-g)] it can be easily proven that \(\psi _2\equiv 0\) and, hence, \(\psi =\psi _1\in span\{\varphi _k,\ldots ,\varphi _{k+h}\}\). \(\square \)

Up to minor changes indicated below, the proof of Lemma 5 is similar to the proof of [31, Proposition 4] so that we sketch the main steps of the proof.

Proof of Lemma 5

Let \(\varphi \in \mathbb {H}_0^{\delta ,s}(\Omega )\), \(\varphi \ne 0\) and \(\lambda >0\) such that

$$\begin{aligned} \langle \varphi , \phi \rangle _{\mathbb {H}_0^{\delta ,s}}=\lambda \int _{\Omega }\varphi \,\phi \,dx\quad \text {for all } \phi \in \mathbb {H}_0^{\delta ,s}(\Omega ). \end{aligned}$$

Up to multiplication by a constant, we can assume that \(\Vert \varphi \Vert _{L^2(\Omega )}^2=\varepsilon \) with \(\varepsilon >0\) to be chosen. Next, for \(k\in \mathbb {N}\), let

$$\begin{aligned} C_k:= 1-2^{-k},\ v_k:= \varphi -C_k,\ w_k:=v_k^+\ \text {and}\ U_k:= \Vert w_k\Vert _{L^2(\Omega )}^2, \end{aligned}$$

where \(v^+:=\max \{v,0\}\). Notice that \(w_k\in \mathbb {H}_0^{\delta ,s}(\Omega )\), since \(\mathbb {H}_0^{\delta ,s}(\Omega )\) is a linear space and \(\varphi (x)-C_k=-C_k\le 0\) a.e. \(x\in \partial _{\delta }\Omega \) so that \(w_k=(-C_k)^+=0\) on \(\partial _{\delta }\Omega \). Moreover, \(0\le w_k\le |\varphi |+|C_k|\le |\varphi |+1\in L^2(\Omega )\) because \(\Omega \) is a bounded domain and \(\lim \nolimits _{k\rightarrow +\infty }w_k=(\varphi -1)^+\). Hence, by the Dominated Convergence Theorem,

$$\begin{aligned} \lim \limits _{k\rightarrow +\infty } U_k=\int _{\Omega }\left[ (\varphi -1)^+\right] ^2dx. \end{aligned}$$
(A.6)

Proceeding in a similar way as in [31, Proposition 4] we can prove that

$$\begin{aligned} \int _{\Omega _{\delta }}\int _{\Omega _{\delta }\cap B(x,\delta )}\frac{|w_{k+1}(x)-w_{k+1}(y)|^2}{|x-y|^{N+2s}}dxdy\le \lambda 2^{k+1} U_k. \end{aligned}$$
(A.7)

In addition, since \(\{w_{k+1}>0\}\subseteq \{w_k>2^{-(k+1)}\}\) for any \(k\in \mathbb {N}\),

$$\begin{aligned} \begin{aligned} U_k&=\Vert w_k\Vert _{L^2(\Omega )}^2\ge \int _{\{w_k>2^{-(k+1)}\}}w_k^2(x)dx\ge 2^{-2(k+1)}|\{w_k>2^{-(k+1)}\}|\\&\ge 2^{-2(k+1)}|\{w_{k+1}>0\}|. \end{aligned} \end{aligned}$$
(A.8)

On the other hand, using Hölder inequality with exponents \(p=\frac{2_s^*}{2}\) and \(q=\frac{N}{2s}\), the fractional Sobolev inequality (cf.[15, Theorem 6.5]) and Proposition 1, we find,

$$\begin{aligned} U_{k+1}\le c\int _{\Omega _{\delta }}\int _{\Omega _{\delta }\cap B(x,\delta )}\frac{|w_{k+1}(x)-w_{k+1}(y)|^2}{|x-y|^{N+2s}}dxdy\left| \{w_{k+1}>0\}\right| ^{\frac{2s}{N}}, \end{aligned}$$

for a positive constant \(c=c(N,s,\delta )\). As a consequence, by (A.7) and (A.8),

$$\begin{aligned} \begin{aligned} U_{k+1}&\le \left( c2^{k+1}\lambda U_k\right) \left( 2^{2(k+1)}U_k\right) ^{\frac{2s}{N}}=2^{1+\frac{4s}{N}}c\lambda \left( 2^{1+\frac{4s}{N}}\right) ^kU_k^{1+\frac{2s}{N}}\\&\le \left[ \left( 1+2^{1+\frac{4s}{N}}c\lambda \right) 2^{1+\frac{4s}{N}}\right] ^kU_k^{1+\frac{2s}{N}}=\theta ^k U_k^{\beta }, \end{aligned} \end{aligned}$$

with \(\beta := 1+\frac{2s}{N}\) and \(\theta =\theta (N,s,\delta ,\lambda )>1\). Next, let us choose \(\varepsilon >0\) such that \(\varepsilon ^{\beta -1}<\frac{1}{\theta ^{1/(\beta -1)}}\) and fix \(\eta \in (\varepsilon ^{\beta -1},\frac{1}{\theta ^{1/(\beta -1)}})\). Let us note that, since \(\theta >1\) and \(\beta >1\), we have \(\eta \in (0,1)\). Arguing inductively, it is easily proved that, for any \(k\in \mathbb {N}\),

$$\begin{aligned} U_k\le \varepsilon \eta ^k. \end{aligned}$$

Then, by (A.6), we get \(\displaystyle \int _{\Omega }\left[ (\varphi -1)^+\right] ^2dx=\lim \limits _{k\rightarrow +\infty }U_k=0\) since \(\eta \in (0,1)\). Thus, \((\varphi -1)^+=0\) a.e. in \(\Omega \) so that \(\varphi \le 1\) a.e. in \(\Omega \). By replacing \(\varphi \) with \(-\varphi \) it follows that \(\Vert \varphi \Vert _{L^{\infty }(\Omega )}\le 1\). Taking in mind the scaling \(\Vert \varphi \Vert _{L^2(\Omega }^2=\varepsilon \), we conclude

$$\begin{aligned} \Vert \varphi \Vert _{L^{\infty }(\Omega )}\le C\Vert \varphi \Vert _{L^2(\Omega )}, \end{aligned}$$

for a constant \(C>0\) depending on \(N,s,\delta \) and \(\lambda \). \(\square \)

Rights and permissions

Reprints and permissions

About this article

Check for updates. Verify currency and authenticity via CrossMark

Cite this article

Bellido, J.C., Ortega, A. A restricted nonlocal operator bridging together the Laplacian and the fractional Laplacian. Calc. Var. 60, 71 (2021). https://doi.org/10.1007/s00526-020-01896-1

Download citation

  • Received:

  • Accepted:

  • Published:

  • DOI: https://doi.org/10.1007/s00526-020-01896-1

Mathematics Subject Classification

Navigation